MED-SURG III: CH 34, 37, 66

Ace your homework & exams now with Quizwiz!

10. To which patients should the nurse teach the Therapeutic Lifestyle Changes diet to reduce the risk of coronary artery disease (CAD)? a. All patients to reduce CAD risk b. Patients who have experienced an MI c. Individuals with two or more risk factors for CAD d. Individuals with a cholesterol level >200 mg/dL (5.2 mmol/L)

10. a. The Therapeutic Lifestyle Changes diet includes recommendations for all people, not just those with risk factors, to decrease the risk for CAD.

10. The nurse is caring for a patient newly admitted with heart failure secondary to dilated cardiomyopathy. Which intervention would be a priority? a. Encourage caregivers to learn CPR. b. Consider a consultation with hospice for palliative care. c. Monitor the patient's response to prescribed medications. d. Arrange for the patient to enter a cardiac rehabilitation pro- gram.

10. c

12. What are manifestations of acute coronary syndrome (ACS) (select all that apply)? a. Dysrhythmia b. Stable angina c. Unstable angina d. ST-segment-elevation myocardial infarction (STEMI) e. Non-ST-segment-elevation myocardial infarction (NSTEMI)

12. c, d, e. Unstable angina, ST-segment-elevation myocardial infarction (STEMI), and non-ST-segment-elevation myocardial infarction (NSTEMI) are conditions that are manifestations of acute coronary syndrome (ACS). The other options are not manifestations of ACS.

18. Which effects contribute to making nitrates the first-line therapy for the treatment of angina (select all that apply)? a. Decrease heart rate (HR) b. Decrease preload c. Decrease afterload d. Dilate coronary arteries e. Prevent thrombosis of plaques f. Decrease myocardial contractility

18. b, c, d. Nitrates decrease preload and afterload to decrease the coronary workload and dilate coronary arteries to increase coronary blood supply. The other options are not attributed to nitrates.

5. The hemodynamic changes the nurse expects to nd a er success- ful initiation of intraaortic balloon pump therapy in a patient with cardiogenic shock include (select all that apply) a. decreased SV. b. decreased SVR. c. decreased PAWP. d. increased diastolic BP. e. decreased myocardial oxygen consumption.

5. b, c, d, e,

6. The purpose of adding PEEP to positive pressure ventilation is to a. increase functional residual capacity and improve oxygenation. b. increase FIO2 in an attempt to wean the patient and avoid O2 toxicity. c. determine if the patient is in synchrony with the ventilator or needs to be paralyzed. d. determine if the patient is able to be weaned and avoid the risk of pneumomediastinum.

6. a,

8. The nurse monitors the patient with positive pressure mechanical ventilation for a. paralytic ileus because pressure on the abdominal contents a ects bowel motility. b. diuresis and sodium depletion because of increased release of atrial natriuretic peptide. c. signs of cardiovascular insu ciency because pressure in the chest impedes venous return. d. respiratory acidosis in a patient with COPD because of alveolar hyperventilation and increased PaO2 levels.

8. c

3. A 52-year-old male patient has received a bolus dose and an infusion of alteplase (Activase) for an ST-segment elevation myocardial infarction (STEMI). To determine the effectiveness of this medication, the nurse should assess the patient for the presence of chest pain. blood in the urine or stool. tachycardia with hypotension. decreased level of consciousness.

A Alteplase is a fibrinolytic that is administered to patients who have had an STEMI. If the medication is effective, the patient's chest pain will resolve because the medication dissolves the thrombus in the coronary artery and results in reperfusion of the myocardium. Bleeding is a major complication of fibrinolytic therapy. Signs of major bleeding include decreased level of consciousness, blood in the urine or stool, and increased heart rate with decreased blood pressure.

34. Which information about a patient who has been receiving thrombolytic therapy for an acute myocardial infarction (AMI) is most important for the nurse to communicate to the health care provider? a. No change in the patient's chest pain b. An increase in troponin levels from baseline c. A large bruise at the patient's IV insertion site d. A decrease in ST-segment elevation on the electrocardiogram

ANS: A Continued chest pain suggests that the thrombolytic therapy is not effective and that other interventions such as percutaneous coronary intervention (PCI) may be needed. Bruising is a possible side effect of thrombolytic therapy, but it is not an indication that therapy should be discontinued. The decrease of the ST-segment elevation indicates that thrombolysis is occurring and perfusion is returning to the injured myocardium. An increase in troponin levels is expected with reperfusion and is related to the washout of cardiac markers into the circulation as the blocked vessel is opened.

27. A patient who is being admitted to the emergency department with intermittent chest pain gives the following list of medications to the nurse. Which medication has the most immediate implications for the patient's care? a. Sildenafil (Viagra) b. Furosemide (Lasix) c. Captopril (Capoten) d. Warfarin (Coumadin)

ANS: A The nurse will need to avoid giving nitrates to the patient because nitrate administration is contraindicated in patients who are using sildenafil because of the risk of severe hypotension caused by vasodilation. The other home medications also should be documented and reported to the health care provider but do not have as immediate an impact on decisions about the patient's treatment.

19. Three days after experiencing a myocardial infarction (MI), a patient who is scheduled for discharge asks for assistance with hygiene activities, saying, "I am too nervous to take care of myself." Based on this information, which nursing diagnosis is appropriate? a. Ineffective coping related to anxiety b. Activity intolerance related to weakness c. Denial related to lack of acceptance of the MI d. Disturbed personal identity related to understanding of illness

ANS: A The patient data indicate that ineffective coping after the MI caused by anxiety about the impact of the MI is a concern. The other nursing diagnoses may be appropriate for some patients after an MI, but the data for this patient do not support denial, activity intolerance, or disturbed personal identity.

Two days after an acute myocardial infarction (MI), a patient complains of stabbing chest pain that increases with a deep breath. Which action will the nurse take first? a. Auscultate the heart sounds. b. Check the patient's temperature. c. Notify the patient's health care provider. d. Give the PRN acetaminophen (Tylenol).

ANS: A The patient's clinical manifestations and history are consistent with pericarditis, and the first action by the nurse should be to listen for a pericardial friction rub. Checking the temperature and notifying the health care provider are also appropriate actions but would not be done before listening for a rub. It is not stated for what symptom (e.g., headache) or finding (e.g., increased temperature) the PRN acetaminophen (Tylenol) is ordered.

39. To improve the physical activity level for a mildly obese 71-year-old patient, which action should the nurse plan to take? a. Stress that weight loss is a major benefit of increased exercise. b. Determine what kind of physical activities the patient usually enjoys. c. Tell the patient that older adults should exercise for no more than 20 minutes at a time. d. Teach the patient to include a short warm-up period at the beginning of physical activity.

ANS: B Because patients are more likely to continue physical activities that they already enjoy, the nurse will plan to ask the patient about preferred activities. The goal for older adults is 30 minutes of moderate activity on most days. Older adults should plan for a longer warm-up period. Benefits of exercises, such as improved activity tolerance, should be emphasized rather than aiming for significant weight loss in older mildly obese adults.

3. Which assessment data collected by the nurse who is admitting a patient with chest pain suggest that the pain is caused by an acute myocardial infarction (AMI)? a. The pain increases with deep breathing. b. The pain has lasted longer than 30 minutes. c. The pain is relieved after the patient takes nitroglycerin. d. The pain is reproducible when the patient raises the arms.

ANS: B Chest pain that lasts for 20 minutes or more is characteristic of AMI. Changes in pain that occur with raising the arms or with deep breathing are more typical of musculoskeletal pain or pericarditis. Stable angina is usually relieved when the patient takes nitroglycerin.

28. Which assessment finding by the nurse caring for a patient who has had coronary artery bypass grafting using a right radial artery graft is most important to communicate to the health care provider? a. Complaints of incisional chest pain b. Pallor and weakness of the right hand c. Fine crackles heard at both lung bases d. Redness on both sides of the sternal incision

ANS: B The changes in the right hand indicate compromised blood flow, which requires immediate evaluation and actions such as prescribed calcium channel blockers or surgery. The other changes are expected and/or require nursing interventions.

13. When titrating IV nitroglycerin (Tridil) for a patient with a myocardial infarction (MI), which action will the nurse take to evaluate the effectiveness of the medication? a. Monitor heart rate. b. Ask about chest pain. c. Check blood pressure. d. Observe for dysrhythmias.

ANS: B The goal of IV nitroglycerin administration in MI is relief of chest pain by improving the balance between myocardial oxygen supply and demand. The nurse also will monitor heart rate and blood pressure (BP) and observe for dysrhythmias, but these parameters will not indicate whether the medication is effective.

40. Which patient at the cardiovascular clinic requires the most immediate action by the nurse? a. Patient with type 2 diabetes whose current blood glucose level is 145 mg/dL b. Patient with stable angina whose chest pain has recently increased in frequency c. Patient with familial hypercholesterolemia and a total cholesterol of 465 mg/dL d. Patient with chronic hypertension whose blood pressure today is 172/98 mm Hg

ANS: B The history of more frequent chest pain suggests that the patient may have unstable angina, which is part of the acute coronary syndrome spectrum. This will require rapid implementation of actions such as cardiac catheterization and possible percutaneous coronary intervention. The data about the other patients suggest that their conditions are stable.

13. An intraaortic balloon pump (IABP) is being used for a patient who is in cardiogenic shock. Which assessment data indicate to the nurse that the goals of treatment with the IABP are being met? a. Urine output of 25 mL/hr b. Heart rate of 110 beats/minute c. Cardiac output (CO) of 5 L/min d. Stroke volume (SV) of 40 mL/beat

ANS: C A CO of 5 L/min is normal and indicates that the IABP has been successful in treating the shock. The low SV signifies continued cardiogenic shock. The tachycardia and low urine output also suggest continued cardiogenic shock.

32. A patient admitted to the coronary care unit (CCU) with an ST-segment-elevation myocardial infarction (STEMI) is restless and anxious. The blood pressure is 86/40 and heart rate is 123. Based on this information, which nursing diagnosis is a priority for the patient? a. Acute pain related to myocardial infarction b. Anxiety related to perceived threat of death c. Stress overload related to acute change in health d. Decreased cardiac output related to cardiogenic shock

ANS: C All the nursing diagnoses may be appropriate for this patient, but the hypotension and tachycardia indicate decreased cardiac output and shock from the damaged myocardium. This will result in decreased perfusion to all vital organs (e.g., brain, kidney, heart) and is a priority.

14. The nurse is caring for a patient who has an intraaortic balloon pump in place. Which action should be included in the plan of care? a. Position the patient supine at all times. b. Avoid the use of anticoagulant medications. c. Measure the patient's urinary output every hour. d. Provide passive range of motion for all extremities.

ANS: C Monitoring urine output will help determine whether the patient's cardiac output has improved and also help monitor for balloon displacement. The head of the bed can be elevated up to 30 degrees. Heparin is used to prevent thrombus formation. Limited movement is allowed for the extremity with the balloon insertion site to prevent displacement of the balloon.

Which statement by a patient with restrictive cardiomyopathy indicates that the nurse's discharge teaching about self-management has been most effective? a. "I will avoid taking aspirin or other antiinflammatory drugs." b. "I will need to limit my intake of salt and fluids even in hot weather." c. "I will take antibiotics when my teeth are cleaned at the dental office." d. "I should begin an exercise program that includes things like biking or swimming."

ANS: C Patients with restrictive cardiomyopathy are at risk for infective endocarditis and should use prophylactic antibiotics for any procedure that may cause bacteremia. The other statements indicate a need for more teaching by the nurse. Dehydration and vigorous exercise impair ventricular filling in patients with restrictive cardiomyopathy. There is no need to avoid salt (unless ordered), aspirin, or NSAIDs.

35. The nurse obtains the following data when assessing a patient who experienced an ST-segment-elevation myocardial infarction (STEMI) 2 days previously. Which information is most important to report to the health care provider? a. The troponin level is elevated. b. The patient denies ever having a heart attack. c. Bilateral crackles are auscultated in the mid-lower lobes. d. The patient has occasional premature atrial contractions (PACs).

ANS: C The crackles indicate that the patient may be developing heart failure, a possible complication of myocardial infarction (MI). The health care provider may need to order medications such as diuretics or angiotensin-converting enzyme (ACE) inhibitors for the patient. Elevation in troponin level at this time is expected. PACs are not life-threatening dysrhythmias. Denial is a common response in the immediate period after the MI.

14. A patient with ST-segment elevation in three contiguous electrocardiographic (ECG) leads is admitted to the emergency department (ED) and diagnosed as having an ST-segment-elevation myocardial infarction (STEMI). Which question should the nurse ask to determine whether the patient is a candidate for thrombolytic therapy? a. "Do you have any allergies?" b. "Do you take aspirin on a daily basis?" c. "What time did your chest pain begin?" d. "Can you rate your chest pain using a 0 to 10 scale?"

ANS: C Thrombolytic therapy should be started within 6 hours of the onset of the myocardial infarction (MI), so the time at which the chest pain started is a major determinant of the appropriateness of this treatment. The other information will also be needed, but it will not be a factor in the decision about thrombolytic therapy.

21. The nurse is obtaining a health history from a 24-year-old patient with hypertrophic cardiomyopathy (HC). Which information obtained by the nurse is most important? a. The patient has a history of a recent upper respiratory infection. b. The patient has a family history of coronary artery disease (CAD). c. The patient reports using cocaine a "couple of times" as a teenager. d. The patient's 29-year-old brother died from a sudden cardiac arrest.

ANS: D About half of all cases of HC have a genetic basis, and it is the most common cause of sudden cardiac death in otherwise healthy young people. The information about the patient's brother will be helpful in planning care (such as an automatic implantable cardioverter-defibrillator [AICD]) for the patient and in counseling other family members. The patient should be counseled against the use of stimulant drugs, but the limited past history indicates that the patient is not at current risk for cocaine use. Viral infections and CAD are risk factors for dilated cardiomyopathy, but not for HC.

15. While waiting for cardiac transplantation, a patient with severe cardiomyopathy has a ventricular assist device (VAD) implanted. When planning care for this patient, the nurse should anticipate a. giving immunosuppressive medications. b. preparing the patient for a permanent VAD. c. teaching the patient the reason for complete bed rest. d. monitoring the surgical incision for signs of infection.

ANS: D The insertion site for the VAD provides a source for transmission of infection to the circulatory system and requires frequent monitoring. Patient's with VADs are able to have some mobility and may not be on bed rest. The VAD is a bridge to transplantation, not a permanent device. Immunosuppression is not necessary for nonbiologic devices like the VAD.

A patient admitted with acute dyspnea is newly diagnosed with dilated cardiomyopathy. Which information will the nurse plan to teach the patient about managing this disorder? a. A heart transplant should be scheduled as soon as possible. b. Elevating the legs above the heart will help relieve dyspnea. c. Careful compliance with diet and medications will prevent heart failure. d. Notify the doctor about any symptoms of heart failure such as shortness of breath.

ANS: D The patient should be instructed to notify the health care provider about any worsening of heart failure symptoms. Because dilated cardiomyopathy does not respond well to therapy, even patients with good compliance with therapy may have recurrent episodes of heart failure. Elevation of the legs above the heart will worsen symptoms (although this approach is appropriate for a patient with hypertrophic cardiomyopathy). The patient with terminal or end-stage cardiomyopathy may consider heart transplantation.

31. When caring for a patient with acute coronary syndrome who has returned to the coronary care unit after having angioplasty with stent placement, the nurse obtains the following assessment data. Which data indicate the need for immediate action by the nurse? a. Heart rate 102 beats/min b. Pedal pulses 1+ bilaterally c. Blood pressure 103/54 mm Hg d. Chest pain level 7 on a 0 to 10 point scale

ANS: D The patient's chest pain indicates that restenosis of the coronary artery may be occurring and requires immediate actions, such as administration of oxygen and nitroglycerin, by the nurse. The other information indicates a need for ongoing assessments by the nurse.

11. The nurse is providing teaching to a patient recovering from a myocardial infarction. How should resumption of sexual activity be discussed? A Delegated to the primary care provider B Discussed along with other physical activities C Avoided because it is embarrassing to the patient D Accomplished by providing the patient with written material

B Although some nurses may not feel comfortable discussing sexual activity with patients, it is a necessary component of patient teaching. It is helpful to consider sex as a physical activity and to discuss or explore feelings in this area when other physical activities are discussed. Although providing the patient with written material is appropriate, it should not replace a verbal dialogue that can address the individual patient's questions and concerns.

1. The nurse prepares a discharge teaching plan for a 44-year-old male patient who has recently been diagnosed with coronary artery disease. Which risk factor should the nurse plan to focus on during the teaching session? Type A personality Elevated serum lipids Family cardiac history Hyperhomocysteinemia

B Dyslipidemia is one of the four major modifiable risk factors for coronary artery disease (CAD). The other major modifiable risk factors are hypertension, tobacco use, and physical inactivity. Research findings related to psychologic states (i.e., type A personality) as a risk factor for coronary artery disease have been inconsistent. Family history is a nonmodifiable risk factor. High homocysteine levels have been linked to an increased risk for CAD.

32. When planning care for the patient with hypertrophic CMP, what should the nurse include? a. Ventricular pacing b. Administration of vasodilators c. Teach the patient to avoid strenuous activity and dehydration d. Surgery for cardiac transplantation will need to be done soon

c. Nursing interventions for the patient with hypertrophic CMP are to improve ventricular filling by reducing ventricular contractility and relieving left ventricular outflow obstruction to relieve symptoms and prevent complications. Strenuous activity and dehydration will increase systemic vascular resistance and should be avoided. Atrioventricular pacing will allow the septum to move away from the left ventricular wall and reduce the degree of outflow obstruction. Vasodilators may decrease venous return and further increase obstruction of blood flow from the heart. The surgery that could be done involves cutting into the thickened septal wall and removing some of the ventricular muscle.

31. What accurately describes dilated CMP (select all that apply)? a. Characterized by ventricular stiffness b. The least common type of cardiomyopathy c. The hyperdynamic systolic function creates a diastolic failure d. Echocardiogram reveals cardiomegaly with thin ventricular walls e. Often follows an infective myocarditis or exposure to toxins or drugs f. Differs from chronic heart failure in that there is no ventricular hypertrophy

d, e, f. Dilated CMP, the most common type of CMP, reveals cardiomegaly with thin ventricular walls on echocardiogram, as there is no ventricular hypertrophy, and may follow an infective myocarditis. As well, stasis of blood in the ventricles may contribute to systemic embolization. Restrictive CMP is the least common type and is characterized by ventricular stiffness. Hypertrophic CMP has hyperdynamic systolic function creating a diastolic failure, is characterized by massive thickening of intraventricular septum and ventricular wall, and may result in syncope during increased activity resulting from an obstructed aortic valve outflow.

18. Which statement about the function of the IABP is accurate? a. Deflation of the balloon allows the HR to increase. b. A primary effect of the IABP is increased systolic blood pressure. c. The rapid deflation of the intraaortic balloon causes a decreased preload. d. During intraaortic counterpulsation, the balloon is inflated during diastole.

d. During intraaortic counterpulsation, the balloon of the IABP is inflated during diastole and deflated during systole. This causes decreased HR, decreased peak systolic pressure, and decreased afterload. (See Table 66-6.)

17. A patient with which disorder would benefit from the use of the intraaortic balloon pump (IABP)? a. An insufficient aortic valve c. Generalized peripheral vascular disease b. A dissecting thoracic aortic aneurysm d. Acute myocardial infarction with cardiogenic shoc

d. The counterpulsation of the intraaortic balloon pump (IABP) increases diastolic arterial pressure, forcing blood back into the coronary arteries and main branches of the aortic arch, increasing coronary artery perfusion pressure and blood flow to the myocardium. The IABP also causes a drop in aortic pressure just before systole, decreasing afterload and myocardial oxygen consumption. These effects make the IABP valuable in treating unstable angina, acute myocardial infarction with cardiogenic shock, and a variety of surgical heart situations. Its use is contraindicated in incompetent aortic valves, dissecting aortic and thoracic aneurysms, and generalized peripheral vascular disease.

35. Collaborative care of the patient with NSTEMI differs from that of a patient with STEMI in that NSTEMI is more frequently initially treated with what? a. PCI c. Acute intensive drug therapy b. CABG d. Reperfusion therapy with thrombolytics

35. c. Because an NSTEMI is an acute coronary syndrome that indicates a transient thrombosis or incomplete coronary artery occlusion, treatment involves intensive drug therapy with antiplatelets, glycoprotein IIb/IIIa inhibitors, antithrombotics, and heparin to prevent clot extension. In addition, IV nitroglycerin is used. Reperfusion therapy using thrombolytics, CABG, or PCI is used for treatment of STEMI.

7. The nursing management of a patient with an arti cial airway includes a. maintaining ET tube cu pressure at 30 cm H2O. b. routine suctioning of the tube at least every 2 hours. c. observing for cardiac dysrhythmias during suctioning. d. preventing tube dislodgment by limiting mouth care to lubrica- tion of the lips.

7. c,

9. Which diagnostic study best di erentiates the various types of cardiomyopathy? a. Echocardiography b. Arterial blood gases c. Cardiac catheterization d. Endomyocardial biopsy

9. a,

23. When providing nutritional counseling for patients at risk for coronary artery disease (CAD), which foods would the nurse encourage patients to include in their diet (select all that apply.)? A Tofu B Walnuts C Tuna fish D Whole milk E Orange juice

A B C Tuna fish, tofu, and walnuts are all rich in omega-3 fatty acids, which have been shown to reduce the risks associated with CAD when consumed regularly.

5. Which patient is at greatest risk for sudden cardiac death (SCD)? A A 42-yr-old white woman with hypertension and dyslipidemia B A 52-yr-old African American man with left ventricular failure C A 62-yr-old obese man with diabetes mellitus and high cholesterol D A 72-yr-old Native American woman with a family history of heart disease

B Patients with left ventricular dysfunction (ejection fraction

4. Which antilipemic medications should the nurse question for a patient with cirrhosis of the liver (select all that apply)? A Niacin (Nicobid) B Ezetimibe (Zetia) C Gemfibrozil (Lopid) D Atorvastatin (Lipitor) E Cholestyramine (Questran)

B D Ezetimibe (Zetia) should not be used by patients with liver impairment. Adverse effects of atorvastatin (Lipitor), a statin drug, include liver damage and myopathy. Liver enzymes must be monitored frequently and the medication stopped if these enzymes increase. Niacin's side effects subside with time, although decreased liver function may occur with high doses. Cholestyramine is safe for long-term us

The most common finding in individuals at risk for sudden cardiac death is A aortic valve disease B mitral valve disease C left ventricular dysfunction D atherosclerotic heart disease

C

30. The patient is admitted post-radiation therapy with symptoms of cardiomyopathy (CMP). Which type of CMP should the nurse suspect that the patient is experiencing? a. Dilated b. Restrictive c. Takotsubo d. Hypertrophic

b. A secondary cause of restrictive cardiomyopathy (CMP) is radiation treatment to the thorax with stiffness of the ventricular wall occurring. Dilated CMP may have a genetic link, follow infectious myocarditis, or be related to an autoimmune process or excess alcohol ingestion. Takotsubo CMP is an acute stress-related syndrome that mimics acute coronary syndrome. It is most common in postmenopausal women. Hypertrophic CMP has a genetic link in about one half of all cases and is frequently seen in young athletic individuals.

36. During treatment with reteplase (Retavase) for a patient with a STEMI, which finding should most concern the nurse? a. Oozing of blood from the IV site b. BP of 102/60 mm Hg with an HR of 78 bpm c. Decrease in the responsiveness of the patient d. Presence of intermittent accelerated idioventricular dysrhythmias

36. c. Decreasing level of consciousness (LOC) may reflect hypoxemia resulting from internal bleeding, which is always a risk with thrombolytic therapy. Oozing of blood is expected, as are reperfusion dysrhythmias. BP is low but not considered abnormal because the pulse is within normal range.

33. When admitting a patient with a non-ST-segment-elevation myocardial infarction (NSTEMI) to the intensive care unit, which action should the nurse perform first? a. Obtain the blood pressure. b. Attach the cardiac monitor. c. Assess the peripheral pulses. d. Auscultate the breath sounds.

ANS: B Because dysrhythmias are the most common complication of myocardial infarction (MI), the first action should be to place the patient on a cardiac monitor. The other actions also are important and should be accomplished as quickly as possible.

30. Which electrocardiographic (ECG) change is most important for the nurse to report to the health care provider when caring for a patient with chest pain? a. Inverted P wave b. Sinus tachycardia c. ST-segment elevation d. First-degree atrioventricular block

ANS: C The patient is likely to be experiencing an ST-segment-elevation myocardial infarction (STEMI). Immediate therapy with percutaneous coronary intervention (PCI) or thrombolytic medication is indicated to minimize myocardial damage. The other ECG changes may also suggest a need for therapy, but not as rapidly.

29. When caring for a patient who has just arrived on the medical-surgical unit after having cardiac catheterization, which nursing intervention should the nurse delegate to a licensed practical/vocational nurse (LPN/LVN)? a. Give the scheduled aspirin and lipid-lowering medication. b. Perform the initial assessment of the catheter insertion site. c. Teach the patient about the usual postprocedure plan of care. d. Titrate the heparin infusion according to the agency protocol.

ANS: A Administration of oral medications is within the scope of practice for LPNs/LVNs. The initial assessment of the patient, patient teaching, and titration of IV anticoagulant medications should be done by the registered nurse (RN).

26. A patient who has recently started taking pravastatin (Pravachol) and niacin (Nicobid) reports the following symptoms to the nurse. Which is most important to communicate to the health care provider? a. Generalized muscle aches and pains b. Dizziness when changing positions quickly c. Nausea when taking the drugs before eating d. Flushing and pruritus after taking the medications

ANS: A Muscle aches and pains may indicate myopathy and rhabdomyolysis, which have caused acute kidney injury and death in some patients who have taken the statin medications. These symptoms indicate that the pravastatin may need to be discontinued. The other symptoms are common side effects when taking niacin, and although the nurse should follow-up with the health care provider, they do not indicate that a change in medication is needed.

10. The nurse will suspect that the patient with stable angina is experiencing a side effect of the prescribed metoprolol (Lopressor) if the a. patient is restless and agitated. b. blood pressure is 90/54 mm Hg. c. patient complains about feeling anxious. d. cardiac monitor shows a heart rate of 61 beats/minute.

ANS: B Patients taking β-adrenergic blockers should be monitored for hypotension and bradycardia. Because this class of medication inhibits the sympathetic nervous system, restlessness, agitation, hypertension, and anxiety will not be side effects.

5. After the nurse has finished teaching a patient about the use of sublingual nitroglycerin (Nitrostat), which patient statement indicates that the teaching has been effective? a. "I can expect some nausea as a side effect of nitroglycerin." b. "I should only take the nitroglycerin if I start to have chest pain." c. "I will call an ambulance if I still have pain after taking 3 nitroglycerin 5 minutes apart." d. "Nitroglycerin helps prevent a clot from forming and blocking blood flow to my heart."

ANS: C The emergency medical services (EMS) system should be activated when chest pain or other symptoms are not completely relieved after 3 sublingual nitroglycerin tablets taken 5 minutes apart. Nitroglycerin can be taken to prevent chest pain or other symptoms from developing (e.g., before intercourse). Gastric upset (e.g., nausea) is not an expected side effect of nitroglycerin. Nitroglycerin does not impact the underlying pathophysiology of coronary artery atherosclerosis.

37. A patient who has chest pain is admitted to the emergency department (ED) and all of the following are ordered. Which one should the nurse arrange to be completed first? a. Chest x-ray b. Troponin level c. Electrocardiogram (ECG) d. Insertion of a peripheral IV

ANS: C The priority for the patient is to determine whether an acute myocardial infarction (AMI) is occurring so that reperfusion therapy can begin as quickly as possible. ECG changes occur very rapidly after coronary artery occlusion, and an ECG should be obtained as soon as possible. Troponin levels will increase after about 3 hours. Data from the chest x-ray may impact the patient's care but are not helpful in determining whether the patient is experiencing a myocardial infarction (MI). Peripheral access will be needed but not before the ECG.

41. A patient with diabetes mellitus and chronic stable angina has a new order for captopril (Capoten). The nurse should teach the patient that the primary purpose of captopril is to a. lower heart rate. b. control blood glucose levels. c. prevent changes in heart muscle. d. reduce the frequency of chest pain.

ANS: C The purpose for angiotensin-converting enzyme (ACE) inhibitors in patients with chronic stable angina who are at high risk for a cardiac event is to decrease ventricular remodeling. ACE inhibitors do not directly impact angina frequency, blood glucose, or heart rate.

6. Which statement made by a patient with coronary artery disease after the nurse has completed teaching about therapeutic lifestyle changes (TLC) diet indicates that further teaching is needed? a. "I will switch from whole milk to 1% milk." b. "I like salmon and I will plan to eat it more often." c. "I can have a glass of wine with dinner if I want one." d. "I will miss being able to eat peanut butter sandwiches."

ANS: D Although only 30% of the daily calories should come from fats, most of the fat in the TLC diet should come from monosaturated fats such as are found in nuts, olive oil, and canola oil. The patient can include peanut butter sandwiches as part of the TLC diet. The other patient comments indicate a good understanding of the TLC diet.

1. When developing a teaching plan for a 61-year-old man with the following risk factors for coronary artery disease (CAD), the nurse should focus on the a. family history of coronary artery disease. b. increased risk associated with the patient's gender. c. increased risk of cardiovascular disease as people age. d. elevation of the patient's low-density lipoprotein (LDL) level.

ANS: D Because family history, gender, and age are nonmodifiable risk factors, the nurse should focus on the patient's LDL level. Decreases in LDL will help reduce the patient's risk for developing CAD.

11. Nadolol (Corgard) is prescribed for a patient with chronic stable angina and left ventricular dysfunction. To determine whether the drug is effective, the nurse will monitor for a. decreased blood pressure and heart rate. b. fewer complaints of having cold hands and feet. c. improvement in the strength of the distal pulses. d. the ability to do daily activities without chest pain.

ANS: D Because the medication is ordered to improve the patient's angina, effectiveness is indicated if the patient is able to accomplish daily activities without chest pain. Blood pressure and heart rate may decrease, but these data do not indicate that the goal of decreased angina has been met. The noncardioselective β-adrenergic blockers can cause peripheral vasoconstriction, so the nurse would not expect an improvement in distal pulse quality or skin temperature.

36. A patient had a non-ST-segment-elevation myocardial infarction (NSTEMI) 3 days ago. Which nursing intervention included in the plan of care is most appropriate for the registered nurse (RN) to delegate to an experienced licensed practical/vocational nurse (LPN/LVN)? a. Evaluation of the patient's response to walking in the hallway b. Completion of the referral form for a home health nurse follow-up c. Education of the patient about the pathophysiology of heart disease d. Reinforcement of teaching about the purpose of prescribed medications

ANS: D LPN/LVN education and scope of practice include reinforcing education that has previously been done by the RN. Evaluating the patient response to exercise after a NSTEMI requires more education and should be done by the RN. Teaching and discharge planning/ documentation are higher level skills that require RN education and scope of practice.

24. A patient with hyperlipidemia has a new order for colesevelam (Welchol). Which nursing action is most appropriate when giving the medication? a. Have the patient take this medication with an aspirin. b. Administer the medication at the patient's usual bedtime. c. Have the patient take the colesevelam with a sip of water. d. Give the patient's other medications 2 hours after the colesevelam.

ANS: D The bile acid sequestrants interfere with the absorption of many other drugs, and giving other medications at the same time should be avoided. Taking an aspirin concurrently with the colesevelam may increase the incidence of gastrointestinal side effects such as heartburn. An increased fluid intake is encouraged for patients taking the bile acid sequestrants to reduce the risk for constipation. For maximum effect, colesevelam should be administered with meals.

38. After receiving change-of-shift report about the following four patients, which patient should the nurse assess first? a. 39-year-old with pericarditis who is complaining of sharp, stabbing chest pain b. 56-year-old with variant angina who is to receive a dose of nifedipine (Procardia) c. 65-year-old who had a myocardial infarction (MI) 4 days ago and is anxious about the planned discharge d. 59-year-old with unstable angina who has just returned to the unit after having a percutaneous coronary intervention (PCI)

ANS: D This patient is at risk for bleeding from the arterial access site for the PCI, so the nurse should assess the patient's blood pressure, pulse, and the access site immediately. The other patients should also be assessed as quickly as possible, but assessment of this patient has the highest priority.

5. Which patient is at greatest risk for sudden cardiac death? A 42-year-old white woman with hypertension and dyslipidemia A 52-year-old African American man with left ventricular failure A 62-year-old obese man with diabetes mellitus and high cholesterol A 72-year-old Native American woman with a family history of heart disease

B Patients with left ventricular dysfunction (ejection fraction < 30%) and ventricular dysrhythmias after MI are at greatest risk for sudden cardiac death (SCD). Other risk factors for SCD include (1) male gender (especially African American men), (2) family history of premature atherosclerosis, (3) tobacco use, (4) diabetes mellitus, (5) hypercholesterolemia, (6) hypertension, and (7) cardiomyopathy.

25. The nurse would assess a patient with complaints of chest pain for which clinical manifestations associated with a myocardial infarction (MI) (select all that apply.)? A Flushing B Ashen skin C Diaphoresis D Nausea and vomiting E S3 or S4 heart sounds

B C D E During the initial phase of an MI, catecholamines are released from the ischemic myocardial cells, causing increased sympathetic nervous system stimulation. This results in the release of glycogen, diaphoresis, and vasoconstriction of peripheral blood vessels. The patient's skin may be ashen, cool, and clammy (not flushed) as a result of this response. Nausea and vomiting may result from reflex stimulation of the vomiting center by severe pain. Ventricular dysfunction resulting from the MI may lead to the presence of the abnormal S3 and S4 heart sounds.

4. A 74-year-old man with a history of prostate cancer and hypertension is admitted to the emergency department with substernal chest pain. Which action will the nurse complete before administering sublingual nitroglycerin? Administer morphine sulfate IV. Auscultate heart and lung sounds. Obtain a 12-lead electrocardiogram (ECG). Assess for coronary artery disease risk factors.

C If a patient has chest pain, the nurse should institute the following measures: (1) administer supplemental oxygen and position the patient in upright position unless contraindicated, (2) assess vital signs, (3) obtain a 12-lead ECG, (4) provide prompt pain relief first with a nitrate followed by an opioid analgesic if needed, and (5) auscultate heart sounds. Obtaining a 12-lead ECG during chest pain aids in the diagnosis.

19. A patient experienced sudden cardiac death (SCD) and survived. Which preventive treatment should the nurse expect to be implemented? A External pacemaker B An electrophysiologic study (EPS) C Medications to prevent dysrhythmias D Implantable cardioverter-defibrillator (ICD)

D An ICD is the most common approach to preventing recurrence of SCD. An external pacemaker may be used in the hospital but will not be used for the patient living daily life at home. An EPS may be done to determine if a recurrence is likely and determine the most effective medication treatment. Medications to prevent dysrhythmias are used but are not the best prevention of SCD.

10. When evaluating a patient's knowledge regarding a low-sodium, low-fat cardiac diet, the nurse recognizes additional teaching is needed when the patient selects which food? A Baked flounder B Angel food cake C Baked potato with margarine D Canned chicken noodle soup

D Canned soups are very high in sodium content. Patients need to be taught to read food labels for sodium and fat content.

2. The nurse instructs a 68-year-old woman with hypercholesterolemia about natural lipid-lowering therapies. The nurse determines further teaching is necessary if the patient makes which statement? "Omega-3 fatty acids are helpful in reducing triglyceride levels." "I should check with my physician before I start taking any herbal products." "Herbal products do not go through as extensive testing as prescription drugs do." "I will take garlic instead of my prescription medication to reduce my cholesterol."

D Current evidence does not support using garlic in the treatment of elevated cholesterol. Strong evidence supports the use of omega-3 fatty acids for reduction of triglyceride levels. Many herbal products are not standardized and effects are not predictable. Patients should consult with their health care provider before starting herbal or natural therapies.

21. A male patient who has coronary artery disease (CAD) has serum lipid values of low-density lipoprotein (LDL) cholesterol of 98 mg/dL and high-density lipoprotein (HDL) cholesterol of 47 mg/dL. What should the nurse include in patient teaching? A Consume a diet low in fats. B Reduce total caloric intake. C Increase intake of olive oil. D The lipid levels are normal.

D For men, the recommended LDL is less than 100 mg/dL, and the recommended level for HDL is greater than 40mg/dL. His normal lipid levels should be included in the patient teaching and encourage him to continue taking care of himself. Assessing his need for teaching related to diet should also be done.

33. When performing discharge teaching for a patient with any type of CMP, what should the nurse instruct the patient to do (select all that apply)? a. Eat a low-sodium diet. b. Go to the gym every day. c. Engage in stress reduction activities. d. Abstain from alcohol and caffeine intake. e. Avoid strenuous activity and allow for periods of rest. f. Suggest that caregivers learn cardiopulmonary resuscitation (CPR).

a, c, d, e, f. These topics can apply to any patient with CMP.

21. What are ventricular assist devices (VADs) designed to do for the patient? a. Provide permanent, total circulatory support when the left ventricle fails b. Partially or totally support circulation temporarily until a donor heart can be obtained c. Support circulation only when patients cannot be weaned from cardiopulmonary bypass d. Reverse the effects of circulatory failure in patients with acute myocardial infarction (MI) in cardiogenic shock

b. Ventricular assist devices (VADs) are temporary devices that can partially or totally support circulation until the heart recovers and can be weaned from cardiopulmonary bypass or until a donor heart can be obtained. The devices currently available do not permanently support circulation.

19. What should the nurse do to prevent arterial trauma during the use of the IABP? a. Reposition the patient every 2 hours. c. Prevent hip flexion of the cannulated leg. b. Check the site for bleeding every hour. d. Cover the insertion site with an occlusive dressing.

c. Because the IABP is inserted into the femoral artery and advanced to the descending thoracic aorta, compromised distal extremity circulation is common and requires that the cannulated extremity be extended at all times. Repositioning the patient to prevent pneumonia is limited to side-lying or supine positions with the head of the bed elevated no more than 30 to 45 degrees. Assessment for bleeding is important because the IABP may cause platelet destruction and occlusive dressings are used to prevent site infection.

20. A patient who is hemodynamically stable has an order to wean the IABP. How should the nurse accomplish this? a. Decrease the augmentation pressure to zero. b. Stop the machine since hemodynamic parameters are satisfactory. c. Stop the infusion flow through the catheter when weaning is initiated. d. Change the pumping ratio from 1:1 to 1:2 or 1:3 until the balloon is removed.

d. Weaning from the IABP involves reducing the pumping to every second or third heartbeat until the IABP catheter is removed. The pumping and infusion flow are continued to reduce the risk for thrombus formation around the catheter until it is removed.

16. Which characteristics describe unstable angina (select all that apply)? a. Usually precipitated by exertion d. Occurs only when the person is recumbent b. Unpredictable and unrelieved by rest e. Usually occurs in response to coronary artery spasm c. Characterized by progressive severity

16. b, c. Unstable angina is unpredictable and unrelieved by rest and has progressively increasing severity. Chronic stable angina is usually precipitated by exertion. Angina decubitus occurs when the person is recumbent. Prinzmetal's angina is frequently caused by a coronary artery spasm.

5. Which characteristics are associated with LDLs (select all that apply)? a. Increases with exercise b. Contains the most cholesterol c. Has an affinity for arterial walls d. Carries lipids away from arteries to liver e. High levels correlate most closely with CAD f. The higher the level, the lower the risk for CAD

5. b, c, e. LDLs contain more cholesterol than the other lipoproteins, have an attraction for arterial walls, and correlate most closely with increased incidence of atherosclerosis and CAD. HDLs increase with exercise and carry lipids away from arteries to the liver for metabolism. A high HDL level is associated with a lower risk of CAD.

6. Which factor should be considered when caring for a woman with suspected coronary artery disease? A Fatigue may be the first symptom. B Classic signs and symptoms are expected. C Increased risk is present before menopause. D Women are more likely to develop collateral circulation.

A Fatigue, rather than pain or shortness of breath, may be the first symptom of impaired cardiac circulation. Women may not exhibit the classic signs and symptoms of ischemia such as chest pain which radiates down the left arm. Neck, throat, or back pain may be symptoms experienced by women. Risk for coronary artery disease increases four times after menopause. Men are more likely to develop collateral circulation.

1. Which patient is most likely to be in the fibrous stage of development of coronary artery disease (CAD)? a. Age 40, thrombus adhered to the coronary artery wall b. Age 50, rapid onset of disease with hypercholesterolemia c. Age 32, thickened coronary arterial walls with narrowed vessel lumen d. Age 19, elevated low-density lipoprotein (LDL) cholesterol, lipid-filled smooth muscle cells

1. c. The fibrous plaque stage has progressive changes that can be seen by age 30. Collagen covers the fatty streak and forms a fibrous plaque in the artery. The thrombus adheres to the arterial wall in the complicated lesion stage. Rapid onset of coronary artery disease (CAD) with hypercholesterolemia may be related to familial hypercholesterolemia, not a stage of CAD development. The fatty streak stage is the earliest stage of atherosclerosis and can be seen by age 15.

11. A 62-year-old woman has prehypertension (BP 142/90 mm Hg) and smokes a pack of cigarettes per day. She has no symptoms of CAD but a recent LDL level was 154 mg/dL (3.98 mmol/L). Based on these findings, the nurse would expect that which treatment plan would be used first for this patient? a. Diet and drug therapy c. Diet therapy and smoking cessation b. Exercise instruction only d. Drug therapy and smoking cessation

11. c. Diet therapy and smoking cessation are indicated for a patient without CAD who has prehypertension and an LDL level ≥130 mg/dL. When the patient's LDL level is ≥160 mg/dL, drug therapy would be added to diet therapy. Because tobacco use is related to increased BP and LDL level, the benefit of smoking cessation is almost immediate. Exercise is indicated to reduce risk factors throughout treatment.

13. Myocardial ischemia occurs as a result of increased oxygen demand and decreased oxygen supply. What factors and disorders result in increased oxygen demand (select all that apply)? a. Hypovolemia or anemia b. Increased cardiac workload with aortic stenosis c. Narrowed coronary arteries from atherosclerosis d. Angina in the patient with atherosclerotic coronary arteries e. Left ventricular hypertrophy caused by chronic hypertension f. Sympathetic nervous system stimulation by drugs, emotions, or exertion

13. b, d, e, f. Increased oxygen demand is caused by increasing the workload of the heart, including left ventricular hypertrophy with hypertension, sympathetic nervous stimulation, and anything precipitating angina. Hypovolemia, anemia, and narrowed coronary arteries contribute to decreased oxygen supply.

14. What causes the pain that occurs with myocardial ischemia? a. Death of myocardial tissue b. Dysrhythmias caused by cellular irritability c. Lactic acid accumulation during anaerobic metabolism d. Elevated pressure in the ventricles and pulmonary vessels

14. c. When the coronary arteries are occluded, contractility ceases after several minutes, depriving the myocardial cells of glucose and oxygen for aerobic metabolism. Anaerobic metabolism begins and lactic acid accumulates, irritating myocardial nerve fibers that then transmit a pain message to the cardiac nerves and upper thoracic posterior roots. The other factors may occur during vessel occlusion but are not the source of pain.

15. What types of angina can occur in the absence of CAD (select all that apply)? a. Silent ischemia d. Microvascular angina b. Nocturnal angina e. Chronic stable angina c. Prinzmetal's angina

15. c, d. Prinzmetal's angina and microvascular angina may occur in the absence of CAD but with arterial spasm in Prinzmetal's angina or abnormalities of the coronary microcirculation. Silent ischemia is prevalent in persons with diabetes mellitus and contributes to asymptomatic myocardial ischemia. Nocturnal angina occurs only at night. Chronic stable angina refers to chest pain that occurs with the same pattern of onset, duration, and intensity intermittently over a long period of time.

17. Tachycardia that is a response of the sympathetic nervous system to the pain of ischemia is detrimental because it increases oxygen demand and a. increases cardiac output. b. causes reflex hypotension. c. may lead to atrial dysrhythmias. d. impairs perfusion of the coronary arteries.

17. d. An increased heart rate (HR) decreases the time the heart spends in diastole, which is the time of greatest coronary blood flow. Unlike other arteries, coronary arteries are perfused when the myocardium relaxes and blood backflows from the aorta into the sinuses of Valsalva, which have openings to the right and left coronary arteries. Thus the heart has a decreased oxygen supply at a time when there is an increased oxygen demand. Tachycardia may also lead to ventricular dysrhythmia. The other options are incorrect.

19. The patient has used sublingual nitroglycerin and various long-acting nitrates but now has an ejection fraction of 38% and is considered at a high risk for a cardiac event. Which medication would first be added for vasodilation and to reduce ventricular remodeling? a. Clopidogrel (Plavix) c. Diltiazem (Cardizem) b. Captopril (Capoten) d. Metoprolol (Lopressor)

19. b. Captopril (Capoten) would be added. It is an angiotensin- converting enzyme (ACE) inhibitor that vasodilates and decreases endothelial dysfunction and may prevent ventricular remodeling. Clopidogrel (Plavix) is an antiplatelet agent used as an alternative for a patient unable to use aspirin. Diltiazem (Cardizem), a calcium channel blocker, may be used to decrease vasospasm but is not known to prevent ventricular remodeling. Metoprolol (Lopressor) is a β-adrenergic blocker that inhibits sympathetic nervous stimulation of the heart.

2. What accurately describes the pathophysiology of CAD? a. Partial or total occlusion of the coronary artery occurs during the stage of raised fibrous plaque b. Endothelial alteration may be caused by chemical irritants such as hyperlipidemia or by tobacco use c. Collateral circulation in the coronary circulation is more likely to be present in the young patient with CAD d. The leading theory of atherogenesis proposes that infection and fatty dietary intake are the basic underlying causes of atherosclerosis

2. b. The etiology of CAD includes atherosclerosis as the major cause. The pathophysiology of atherosclerosis development is related to endothelial chemical injury and inflammation, which can be the result of tobacco use, hyperlipidemia, hypertension, toxins, diabetes mellitus, hyperhomocysteinemia, and infection causing a local inflammatory response in the inner lining of the vessel walls. Partial or total occlusion occurs in the complicated lesion stage. Extra collateral circulation occurs in the presence of chronic ischemia. Therefore it is more likely to occur in an older patient.

20. When instructing the patient with angina about taking sublingual nitroglycerin tablets, what should the nurse teach the patient? a. To lie or sit and place one tablet under the tongue when chest pain occurs b. To take the tablet with a large amount of water so it will dissolve right away c. That if one tablet does not relieve the pain in 15 minutes, the patient should go to the hospital d. That if the tablet causes dizziness and a headache, stop the medication and call the doctor or go to the hospital

20. a. A common complication of nitrates is dizziness caused by orthostatic hypotension, so the patient should sit or lie down and place the tablet under the tongue. The tablet should be allowed to dissolve under the tongue. To prevent the tablet from being swallowed, water should not be taken with it. The recommended dose for the patient for whom nitroglycerin (NTG) has been prescribed is one tablet taken sublingually (SL) or one metered spray for symptoms of angina. If symptoms are unchanged or worse after 5 minutes, the patient should contact the emergency medical services (EMS) system before taking additional NTG. If symptoms are significantly improved by one dose of NTG, instruct the patient or caregiver to repeat NTG every 5 minutes for a maximum of three doses and contact EMS if symptoms have not resolved completely. Headache is also a common complication of nitrates but usually resolves with continued use of nitrates and may be controlled with mild analgesics.

21. When teaching an older adult with CAD how to manage the treatment program for angina, which guidelines does the nurse use to teach the patient? a. To sit for 2 to 5 minutes before standing when getting out of bed b. To exercise only twice a week to avoid unnecessary strain on the heart c. That lifestyle changes are not as necessary as they would be in a younger person d. That aspirin therapy is contraindicated in older adults because of the risk for bleeding

21. a. Orthostatic hypotension may cause dizziness and falls in older adults taking antianginal agents that decrease preload. Patients should be cautioned to change positions slowly. Daily exercise programs are indicated for older adults and may increase performance, endurance, and ability to tolerate stress. A change in lifestyle behaviors may increase the quality of life and reduce the risks of CAD, even in the older adult. Aspirin is commonly used in these patients and is not contraindicated.

22. When a patient reports chest pain, why must unstable angina be identified and rapidly treated? a. The pain may be severe and disabling. b. ECG changes and dysrhythmias may occur during an attack. c. Atherosclerotic plaque deterioration may cause complete thrombus of the vessel lumen. d. Spasm of a major coronary artery may cause total occlusion of the vessel with progression to MI.

22. c. Unstable angina is associated with the rupture of a once-stable atherosclerotic plaque, exposing the intima to blood and stimulating platelet aggregation and local vasoconstriction with thrombus formation. Patients with unstable angina require immediate hospitalization and monitoring because the lesion is at increased risk of complete thrombosis of the lumen with progression to MI. Any type of angina may be associated with severe pain, ECG changes, and dysrhythmias. Prinzmetal's angina is characterized by coronary artery spasm.

23. The nurse suspects stable angina rather than MI pain in the patient who reports that his chest pain a. is relieved by nitroglycerin. c. does not radiate to the neck, back, or arms. b. is a sensation of tightness or squeezing. d. is precipitated by physical or emotional exertion.

23. a. One of the primary differences between the pain of angina and the pain of an MI is that angina pain is usually relieved by rest or nitroglycerin, which reduces the oxygen demand of the heart, whereas MI pain is not. Both angina and MI pain can cause a pressure or squeezing sensation; may or may not radiate to the neck, back, arms, fingers, and jaw; and may be precipitated by exertion.

24. A patient admitted to the hospital for evaluation of chest pain has no abnormal serum cardiac markers 4 hours after the onset of pain. What noninvasive diagnostic test can be used to differentiate angina from other types of chest pain? a. 12-lead ECG c. Coronary angiogram b. Exercise stress test d. Transesophageal echocardiogram

24. b. An exercise stress test will reveal ECG changes that indicate impaired coronary circulation when the oxygen demand of the heart is increased. A single ECG is not conclusive for CAD and negative findings do not rule out CAD. Coronary angiography can detect narrowing of coronary arteries but is an invasive procedure. Echocardiograms of various types may identify abnormalities of myocardial wall motion under stress but are indirect measures of CAD.

25. A 52-year-old man is admitted to the emergency department with severe chest pain. On what basis would the nurse suspect an MI? a. He has pale, cool, clammy skin. b. He reports nausea and vomited once at home. c. He says he is anxious and has a feeling of impending doom. d. He reports he has had no relief of the pain with rest or position change.

25. d. The subjective report of the pain from an MI is usually severe. It usually is unrelieved by nitroglycerin, rest, or position change and usually lasts more than the 15 or 20 minutes typical of angina pain. All of the other symptoms may occur with angina as well as with an MI.

26. At what point in the healing process of the myocardium following an infarct does early scar tissue result in an unstable heart wall? a. 2 to 3 days after MI c. 10 to 14 days after MI b. 4 to 10 days after MI d. 6 weeks after MI

26. c. At 10 to 14 days after MI, the myocardium is considered especially vulnerable to increased stress because of the unstable state of healing at this point, as well as the increasing physical activity of the patient. At 2 to 3 days, removal of necrotic tissue is taking place by phagocytic cells. By 4 to 10 days, the necrotic tissue has been cleared and a collagen matrix for scar tissue has been deposited. Healing with scar-tissue replacement of the necrotic area is usually complete by 6 weeks.

27. To detect and treat the most common complication of MI, what should the nurse do? a. Measure hourly urine output. c. Use continuous cardiac monitoring. b. Auscultate the chest for crackles. d. Take vital signs every 2 hours for the first 8 hours.

27. c. The most common complication of MI is cardiac dysrhythmias. Continuous cardiac monitoring allows identification and treatment of dysrhythmias that may cause further deterioration of the cardiovascular status or death. Measurement of hourly urine output and vital signs is indicated to detect symptoms of the complication of cardiogenic shock. Crackles, dyspnea, and tachycardia may indicate the onset of heart failure.

28. During the assessment, the nurse identifies crackles in the lungs and an S3 heart sound. Which complication of MI should the nurse suspect and further investigate? a. Pericarditis c. Ventricular aneurysm b. Heart failure d. Papillary muscle dysfunction

28. b. Heart failure, which can escalate to cardiogenic shock, initially occurs with mild dyspnea, restlessness, agitation, pulmonary congestion with crackles, S3 or S4 heart sounds, and jugular vein distention. Pericarditis is a common complication identified with chest pain that is aggravated by inspiration, coughing, and moving the upper body. Ventricular aneurysm is manifested with heart failure, dysrhythmias, and angina. Papillary muscle dysfunction is suspected with a new systolic apical murmur.

29. In the patient with chest pain, which results can differentiate unstable angina from an MI? a. ECG changes present at the onset of the pain b. A chest x-ray indicating left ventricular hypertrophy c. Appearance of troponin in the blood 48 hours after the infarct d. Creatine kinase (CK)-MB enzyme elevations that peak 18 hours after the infarct

29. d. Creatine kinase-muscle and brain subunits band (CK- MB) is a tissue enzyme that is specific to cardiac muscle and is released into the blood when myocardial cells die. CK-MB levels begin to rise about 6 hours after an acute MI, peak in about 18 hours, and return to normal within 24 to 36 hours. This increase can identify the presence of and quantify myocardiac damage. Cardiac troponin T and troponin I are released with myocardial damage, rise as quickly as CK-MB does, and remain elevated for 2 weeks. ECG changes are often not apparent immediately after infarct and may be normal when the patient seeks medical attention. An enlarged heart, determined by x-ray, indicates cardiac stress but is not diagnostic of acute MI.

3. While obtaining patient histories, which patient does the nurse identify as having the highest risk for CAD? a. A white man, age 54, who is a smoker and has a stressful lifestyle b. A white woman, age 68, with a BP of 172/100 mm Hg and who is physically inactive c. An Asian woman, age 45, with a cholesterol level of 240 mg/dL and a BP of 130/74 mm Hg d. An obese African American man, age 65, with a cholesterol level of 195 mg/dL and a BP of 128/76 mm Hg

3. b. This white woman has one unmodifiable risk factor (age) and two major modifiable risk factors (hypertension and physical inactivity). Her gender risk is as high as a man's because she is over 65 years of age. The white man has one unmodifiable risk factor (gender), one major modifiable risk factor (smoking), and one minor modifiable risk factor (stressful lifestyle). The Asian woman has only one major modifiable risk factor (hyperlipidemia) and Asians in the United States have fewer myocardial infarctions (MIs) than do whites. The African American man has an unmodifiable risk factor related to age and one major modifiable risk factor (obesity).

30. A second 12-lead ECG performed on a patient 4 hours after the onset of chest pain reveals ST segment elevation. What does the nurse recognize that this finding indicates? a. Transient ischemia typical of unstable angina b. Lack of permanent damage to myocardial cells c. MI associated with prolonged and complete coronary thrombosis d. MI associated with transient or incomplete coronary artery occlusion

30. c. A differentiation is made between MIs that have ST segment elevations on ECG and those that do not because chest pain accompanied by ST segment elevations is associated with prolonged and complete coronary thrombosis and is treated with reperfusion therapy. The other options are incorrect.

31. What describes transmyocardial laser revascularization (TMR)? a. Structure applied to hold vessels open b. Requires anticoagulation following the procedure c. Laser-created channels between left ventricular cavity and coronary circulation d. Surgical construction of new vessels to carry blood beyond obstructed coronary artery

31. c. Transmyocardial laser revascularization (TMR) is a treatment used for patients with inoperable CAD. It uses a high-energy laser to create channels in the heart to allow blood to flow to the ischemic area and can be done percutaneously or during surgery with a left anterior thoracotomy incision. A stent is the structure used to hold vessels open and requires anticoagulation following the procedure. Surgical construction of new vessels is done with a coronary artery bypass graft (CABG).

32. Which treatment is used first for the patient with a confirmed MI to open the blocked artery within 90 minutes of arrival to the facility? a. Stent placement c. Percutaneous coronary intervention (PCI) b. Coronary artery bypass graft (CABG) d. Transmyocardial laser revascularization (TMR)

32. c. Emergent percutaneous coronary intervention (PCI) is the first treatment for patients with a confirmed MI within 90 minutes of arriving at the facility with an interventional cardiac catheterization lab. Stent placement, CABG, and TMR are usually done to facilitate circulation in non- emergency situations.

33. Delegation Decision: In planning care for a patient who has just returned to the unit following a PCI, the nurse may delegate which activity to unlicensed assistive personnel (UAP)? a. Monitor the IV fluids and measure urine output. b. Check vital signs and report changes in HR, BP, or pulse oximetry. c. Explain to the patient the need for frequent vital signs and pulse checks. d. Assess circulation to the extremity used by checking pulses, skin temperature, and color.

33. b. Unlicensed assistive personnel (UAP) can check vital signs and report results to the RN. The other actions include assessment, teaching, and monitoring of IV fluids, which are all responsibilities of the RN.

34. A patient is scheduled to have CABG surgery. What does the nurse explain to him that is involved with the procedure? a. A synthetic graft will be used as a tube for blood flow from the aorta to a coronary artery distal to an obstruction. b. A stenosed coronary artery will be resected and a synthetic arterial tube graft will be inserted to replace the diseased artery. c. The internal mammary artery will be detached from the chest wall and attached to a coronary artery distal to the stenosis. d. Reversed segments of a saphenous artery from the aorta will be anastomosed to the coronary artery distal to an obstruction.

34. c. The most common method of coronary artery bypass involves leaving the internal mammary artery attached to its origin from the subclavian artery but dissecting it from the chest wall and anastomosing it distal to an obstruction in a coronary artery. Synthetic grafts are not commonly used as coronary bypass grafts, although research continues to investigate this option. Saphenous veins are used for bypass grafts when additional conduits are needed.

37. The nurse recognizes that thrombolytic therapy for the treatment of an MI has not been successful when the patient displays which manifestation? a. Continues to have chest pain b. Has a marked increase in CK enzyme levels within 3 hours of therapy c. Develops major gastrointestinal (GI) or genitourinary (GU) bleeding during treatment d. Develops premature ventricular contractions and ventricular tachycardia during treatment

37. a. If chest pain is unchanged, it is an indication that reperfusion was not successful. Indications that the occluded coronary artery is patent and blood flow to the myocardium is reestablished following thrombolytic therapy include return of ST segment to baseline on the ECG; relief of chest pain; marked, rapid rise of the CK enzyme within 3 hours of therapy; and the presence of reperfusion dysrhythmias.

38. When the patient who is diagnosed with an MI is not relieved of chest pain with IV nitroglycerin, which medication will the nurse expect to be used? a. IV morphine sulfate c. IV amiodarone (Cordarone) b. Calcium channel blockers d. Angiotensin-converting enzyme (ACE) inhibitors

38. a. Morphine sulfate decreases anxiety and cardiac workload as a vasodilator and reduces preload and myocardial O2 consumption, which relieves chest pain. Calcium channel blockers, amiodarone, and ACE inhibitors will not relieve chest pain related to an MI.

39. What is the rationale for using docusate sodium (Colace) for a patient after an MI? a. Controls ventricular dysrhythmias c. Minimizes bradycardia from vagal stimulation b. Relieves anxiety and cardiac workload d. Prevents the binding of fibrinogen to platelets

39. c. Docusate sodium (Colace) is a stool softener, which prevents straining and provoking dysrhythmias. It does not do any of the other options. Antidysrhythmics are used to control ventricular dysrhythmias; morphine sulfate is used to decrease cardiac workload and anxiety; and glycoprotein IIb/IIIa inhibitors and antiplatelets prevent the binding of fibrinogen to platelets.

4. Priority Decision: While teaching women about the risks and incidence of CAD, what does the nurse emphasize? a. Smoking is not as significant a risk factor for CAD in women as it is in men. b. Women seek treatment sooner than men when they have symptoms of CAD. c. Estrogen replacement therapy in postmenopausal women decreases the risk for CAD. d. CAD is the leading cause of death in women, with a higher mortality rate after MI than in men.

4. d. CAD is the number-one killer of American women and women have a much higher mortality rate within 1 year following MI than do men. Smoking carries specific problems for women because smoking has been linked to a decrease in estrogen levels and to early menopause and it has been identified as the most powerful contributor to CAD in women under the age of 50. Fewer women than men present with classic manifestations and women delay seeking care longer than men. Recent research indicates that estrogen replacement does not reduce the risk for CAD, even though estrogen lowers low-density lipoprotein (LDL) and raises high-density lipoprotein (HDL) cholesterol.

40. The patient has hypertension and just experienced an MI. Which type of medication would be expected to be added to decrease the workload on his heart? a. ACE inhibitor c. Calcium channel blocker b. β-adrenergic blocker d. Angiotensin II receptor blocker (ARB)

40. b. It is recommended that patients with hypertension and after an MI be on β-adrenergic blockers indefinitely to decrease oxygen demand. They inhibit sympathetic nervous stimulation of the heart; reduce heart rate, contractility, and blood pressure; and decrease afterload. Although calcium channel blockers decrease heart rate, contractility, and blood pressure, they are not used unless the patient cannot tolerate β-adrenergic blockers. ACE inhibitors and angiotensin II receptor blockers (ARBs) are used for vasodilation.

41. A patient with an MI is exhibiting anxiety while being taught about possible lifestyle changes. The nurse evaluates that the anxiety is relieved when the patient states a. "I'm going to take this recovery one step at a time." b. "I feel much better and am ready to get on with my life." c. "How soon do you think I will be able to go back to work?" d. "I know you are doing everything possible to save my life."

41. a. This patient is indicating positive coping with a realization that recovery takes time and that lifestyle changes can be made as needed. The patient who is "just going to get on with life" is probably in denial about the seriousness of the condition and the changes that need to be made. Nervous questioning about the expected duration and effect of the condition indicates the presence of anxiety, as does the statement regarding the health care professional's role in treatment.

43. Which statement indicates the patient is experiencing anger as the psychologic response to his acute MI? a. "Yes, I'm having a little chest pain. It's no big deal." b. "I don't think I can take care of myself at home yet." c. "What's going to happen if I have another heart attack?" d. "I hope my wife is happy now after harping at me about my eating habits all these years."

43. d. Anger about the MI may be directed at family, staff, or the medical regimen. Stating that the chest pain is no big deal is denial. Relaying an inability to care for self relates to dependency. Questioning what will happen if there is another attack is expressing anxiety and fear. Depression may be expressed related to changes in lifestyle. Realistic acceptance is seen with actively engaging in changing modifiable risk factors.

44. The nurse and patient set a patient outcome that at the time of discharge after an MI the patient will be able to tolerate moderate-energy activities that are similar to which activity? a. Golfing c. Cycling at 13 mph b. Walking at 5 mph d. Mowing the lawn by hand

44. a. Golfing is a moderate-energy activity that expends about 5 metabolic equivalent units (METs) and is within the 3 to 5 METs activity level desired for a patient by the time of discharge from the hospital following an MI. Walking at 5 mph and mowing the lawn by hand are high-energy activities and cycling at 13 mph is an extremely high-energy activity.

5. A 58-year-old patient is in a cardiac rehabilitation program. The nurse teaches the patient to stop exercising if what occurs? a. Pain or dyspnea develop c. The respiratory rate increases to 30 b. The HR exceeds 150 bpm d. The HR is 30 bpm over the resting HR

45. a. Any activity or exercise that causes dyspnea and chest pain should be stopped in the patient with CAD. The training target for a healthy 58-year-old is 80% of maximum HR, or 130 bpm. In a patient with cardiac disease undergoing cardiac conditioning, however, the HR should not exceed 20 bpm over the resting pulse rate. HR, rather than respiratory rate, determines the parameters for exercise.

46. In counseling the patient about sexual activity following an MI, what should the nurse do? a. Wait for the patient to ask about resuming sexual activity b. Discuss sexual activity while teaching about other physical activity c. Have the patient ask the health care provider when sexual activity can be resumed d. Inform the patient that impotence is a common long-term complication following MI

46. b. Resumption of sexual activity is often difficult for patients to approach and it is reported that most cardiac patients do not resume sexual activity after MI. The nurse can give the patient permission to discuss concerns about sexual activity by introducing it as a physical activity when other physical activities are discussed. Health care providers may have preferences regarding the timing of resumption of sexual activity and the nurse should discuss this with the health care provider and the patient but addressing the patient's concerns is a nursing responsibility. Patients should be informed that impotence after MI is common but that it usually disappears after several attempts.

47. What advice about sexual activity should the nurse give to a male patient who has had an MI? a. The patient should use the superior position. b. Foreplay may cause too great an increase in heart rate. c. Prophylactic nitroglycerin may be used if angina occurs. d. Performance can be enhanced with the use of sildenafil (Viagra).

47. c. It is not uncommon for a patient who experiences chest pain on exertion to have some angina during sexual stimulation or intercourse and the patient should be instructed to use nitroglycerin prophylactically. Positions during intercourse are a matter of individual choice and foreplay is desirable because it allows a gradual increase in HR. Sildenafil (Viagra) should be used cautiously in men with CAD and should not be used with nitrates.

48. Priority Decision: A patient is hospitalized after a successful resuscitation of an episode of sudden cardiac death (SCD). During the care of the patient, what nursing intervention is most important? a. Continuous ECG monitoring c. Frequent assessment of heart sounds b. Auscultation of the carotid arteries d. Monitoring of airway status and respiratory patterns

48. a. Most patients who experience sudden cardiac death (SCD) as a result of CAD do not have an acute MI but have dysrhythmias that cause death, probably as a result of electrical instability of the myocardium. To identify and treat those specific dysrhythmias, continuous monitoring is important. The other assessments can be done but are not the most important after an episode of SCD.

Which serum lipid elevation, along with elevated LDL, is strongly associated with CAD? a. Apolipoproteins b. Fasting triglycerides c. Total serum cholesterol d. High-density lipoprotein (HDL)

6. b. Elevated fasting triglyceride levels are associated with cardiovascular disease and diabetes. Apolipoproteins are found in varying amounts on the HDLs and activate enzyme or receptor sites that promote removal of fat from plasma, which is protective. The apolipoprotein A and apolipropotein B ratio must be done to predict CAD. Elevated HDLs are associated with a lower risk of CAD. Elevated total serum cholesterol must be calculated with HDL for a ratio over time to determine an increased risk of CAD.

7. The laboratory tests for four patients show the following results. Which patient should the nurse teach first about preventing CAD because the patient is at the greatest risk for CAD even without other risk factors? a. Total cholesterol: 152 mg/dL, triglycerides: 148 mg/dL, LDL: 148 mg/dL, HDL: 52 mg/dL b. Total cholesterol: 160 mg/dL, triglycerides: 102 mg/dL, LDL: 138 mg/dL, HDL: 56 mg/dL c. Total cholesterol: 200 mg/dL, triglycerides: 150 mg/dL, LDL: 160 mg/dL, HDL: 48 mg/dL d. Total cholesterol: 250 mg/dL, triglycerides: 164 mg/dL, LDL: 172 mg/dL, HDL: 32 mg/dL

7. d. All of this patient's results are abnormal. The patient in option c is close to being at risk, as all of that patient's results are at or near the cutoff for being acceptable. If this patient is a woman, the HDL is too low. The other patients' results are at acceptable levels.

8. The nurse is encouraging a sedentary patient with major risks for CAD to perform physical exercise on a regular basis. In addition to decreasing the risk factor of physical inactivity, the nurse tells the patient that exercise will also directly contribute to reducing which risk factors? a. Hyperlipidemia and obesity c. Elevated serum lipids and stressful lifestyle b. Diabetes mellitus and hypertension d. Hypertension and elevated serum homocysteine

8. a. Increased exercise without an increase in caloric intake will result in weight loss, reducing the risk associated with obesity. Exercise increases lipid metabolism and increases HDL, thus reducing CAD risk. Exercise may also indirectly reduce the risk of CAD by controlling hypertension, promoting glucose metabolism in diabetes, and reducing stress. Although research is needed to determine whether a decline in homocysteine can reduce the risk of heart disease, it appears that dietary modifications are indicated for risk reduction.

9. During a routine health examination, a 48-year-old patient is found to have a total cholesterol level of 224 mg/dL (5.8 mmol/L) and an LDL level of 140 mg/dL (3.6 mmol/L). What does the nurse teach the patient based on the Therapeutic Lifestyle Changes diet (select all that apply)? a. Use fat-free milk d. Eliminate intake of simple sugars b. Abstain from alcohol use e. Avoid egg yolks and foods prepared with whole eggs c. Reduce red meat in the diet

9. a, c, e. Therapeutic Lifestyle Changes diet recommendations emphasize reduction in saturated fat and cholesterol intake. Red meats, whole milk products, and eggs as well as butter, stick margarine, lard, and solid shortening should be reduced or eliminated from diets. If triglyceride levels are high, alcohol and simple sugars should be reduced.

3. A 52-yr-old male patient has received a bolus dose and an infusion of alteplase (Activase) for an ST-segment elevation myocardial infarction (STEMI). Which patient assessment would determine the effectiveness of the medication? A Presence of chest pain B Blood in the urine or stool C Tachycardia with hypotension D Decreased level of consciousness

A Alteplase is a fibrinolytic agent that is administered to patients who have had an STEMI. If the medication is effective, the patient's chest pain will resolve because the medication dissolves the thrombus in the coronary artery and results in reperfusion of the myocardium. Bleeding is a major complication of fibrinolytic therapy. Signs of major bleeding include decreased level of consciousness, blood in the urine or stool, and increased heart rate with decreased blood pressure.

7. Which individuals would the nurse identify as having the highest risk for coronary artery disease (CAD)? A A 45-yr-old depressed man with a high-stress job B A 60-yr-old man with below normal homocysteine levels C A 54-yr-old woman vegetarian with elevated high-density lipoprotein (HDL) levels D A 62-yr-old woman who has a sedentary lifestyle and body mass index (BMI) of 23 kg/m2

A The 45-yr-old depressed man with a high-stress job is at the highest risk for CAD. Studies demonstrate that depression and stressful states can contribute to the development of CAD. Elevated HDL levels and low homocysteine levels actually help to prevent CAD. Although a sedentary lifestyle is a risk factor, a BMI of 23 kg/m2 depicts normal weight, and thus the patient with two risk factors is at greatest risk for developing CAD.

7. When planning emergent care for a patient with a suspected MI, what should the nurse anticipate administrating? A Oxygen, nitroglycerin, aspirin, and morphine B Oxygen, furosemide (Lasix), nitroglycerin, and meperidine C Aspirin, nitroprusside (Nipride), dopamine (Intropin), and oxygen D Nitroglycerin, lorazepam (Ativan), oxygen, and warfarin (Coumadin)

A The American Heart Association's guidelines for emergency care of the patient with chest pain include the administration of oxygen, nitroglycerin, aspirin, and morphine. These interventions serve to relieve chest pain, improve oxygenation, decrease myocardial workload, and prevent further platelet aggregation. The other medications may be used later in the patient's treatment.

9. When planning emergent care for a patient with a suspected myocardial infarction (MI), what should the nurse anticipate administrating? A Oxygen, nitroglycerin, aspirin, and morphine B Aspirin, nitroprusside, dopamine, and oxygen C Oxygen, furosemide (Lasix), nitroglycerin, and meperidine D Nitroglycerin, lorazepam (Ativan), oxygen, and warfarin (Coumadin)

A The American Heart Association's guidelines for emergency care of the patient with chest pain include the administration of oxygen, nitroglycerin, aspirin, and morphine. These interventions serve to relieve chest pain, improve oxygenation, decrease myocardial workload, and prevent further platelet aggregation. The other medications may be used later in the patient's treatment.

1. The community health nurse is planning health promotion teaching targeted at preventing coronary artery disease (CAD). Which ethnic group would the nurse select as the highest priority for this intervention? A White male B Hispanic male C African American male D Native American female

A The incidence of CAD and myocardial infarction (MI) is highest among white, middle-aged men. Hispanic individuals have lower rates of CAD than non-Hispanic whites or African Americans. African Americans have an earlier age of onset and more severe CAD than whites and more than twice the mortality rate of whites of the same age. Native Americans have increased mortality in less than 35-year-olds and have major modifiable risk factors such as diabetes.

25. The nurse is caring for a patient who was admitted to the coronary care unit following an acute myocardial infarction (AMI) and percutaneous coronary intervention the previous day. Teaching for this patient would include a. when cardiac rehabilitation will begin. b. the typical emotional responses to AMI. c. information regarding discharge medications. d. the pathophysiology of coronary artery disease.

ANS: A Early after an AMI, the patient will want to know when resumption of usual activities can be expected. At this time, the patient's anxiety level or denial will interfere with good understanding of complex information such as the pathophysiology of coronary artery disease (CAD). Teaching about discharge medications should be done closer to discharge. The nurse should support the patient by decreasing anxiety rather than discussing the typical emotional responses to myocardial infarction (MI).

21. A few days after experiencing a myocardial infarction (MI) and successful percutaneous coronary intervention, the patient states, "I just had a little chest pain. As soon as I get out of here, I'm going for my vacation as planned." Which reply would be most appropriate for the nurse to make? a. "What do you think caused your chest pain?" b. "Where are you planning to go for your vacation?" c. "Sometimes plans need to change after a heart attack." d. "Recovery from a heart attack takes at least a few weeks."

ANS: A When the patient is experiencing denial, the nurse should assist the patient in testing reality until the patient has progressed beyond this step of the emotional adjustment to MI. Asking the patient about vacation plans reinforces the patient's plan, which is not appropriate in the immediate post-MI period. Reminding the patient in denial about the MI is likely to make the patient angry and lead to distrust of the nursing staff.

20. When caring for a patient who is recovering from a sudden cardiac death (SCD) event and has no evidence of an acute myocardial infarction (AMI), the nurse will anticipate teaching the patient that a. sudden cardiac death events rarely reoccur. b. additional diagnostic testing will be required. c. long-term anticoagulation therapy will be needed. d. limited physical activity after discharge will be needed to prevent future events.

ANS: B Diagnostic testing (e.g., stress test, Holter monitor, electrophysiologic studies, cardiac catheterization) is used to determine the possible cause of the SCD and treatment options. SCD is likely to recur. Anticoagulation therapy will not have any effect on the incidence of SCD, and SCD can occur even when the patient is resting.

7. After the nurse teaches the patient about the use of carvedilol (Coreg) in preventing anginal episodes, which statement by a patient indicates that the teaching has been effective? a. "Carvedilol will help my heart muscle work harder." b. "It is important not to suddenly stop taking the carvedilol." c. "I can expect to feel short of breath when taking carvedilol." d. "Carvedilol will increase the blood flow to my heart muscle."

ANS: B Patients who have been taking β-adrenergic blockers can develop intense and frequent angina if the medication is suddenly discontinued. Carvedilol (Coreg) decreases myocardial contractility. Shortness of breath that occurs when taking β-adrenergic blockers for angina may be due to bronchospasm and should be reported to the health care provider. Carvedilol works by decreasing myocardial oxygen demand, not by increasing blood flow to the coronary arteries.

9. Diltiazem (Cardizem) is ordered for a patient with newly diagnosed Prinzmetal's (variant) angina. When teaching the patient, the nurse will include the information that diltiazem will a. reduce heart palpitations. b. decrease spasm of the coronary arteries. c. increase the force of the heart contractions. d. help prevent plaque from forming in the coronary arteries.

ANS: B Prinzmetal's angina is caused by coronary artery spasm. Calcium channel blockers (e.g., diltiazem, amlodipine [Norvasc]) are a first-line therapy for this type of angina. Lipid-lowering drugs help reduce atherosclerosis (i.e., plaque formation), and β-adrenergic blockers decrease sympathetic stimulation of the heart (i.e., palpitations). Medications or activities that increase myocardial contractility will increase the incidence of angina by increasing oxygen demand.

23. A patient who is recovering from an acute myocardial infarction (AMI) asks the nurse about when sexual intercourse can be resumed. Which response by the nurse is best? a. "Most patients are able to enjoy intercourse without any complications." b. "Sexual activity uses about as much energy as climbing two flights of stairs." c. "The doctor will provide sexual guidelines when your heart is strong enough." d. "Holding and cuddling are good ways to maintain intimacy after a heart attack."

ANS: B Sexual activity places about as much physical stress on the cardiovascular system as most moderate-energy activities such as climbing two flights of stairs. The other responses do not directly address the patient's question or may not be accurate for this patient.

22. When evaluating the effectiveness of preoperative teaching with a patient scheduled for coronary artery bypass graft (CABG) surgery using the internal mammary artery, the nurse determines that additional teaching is needed when the patient says which of the following? a. "They will circulate my blood with a machine during the surgery." b. "I will have small incisions in my leg where they will remove the vein." c. "They will use an artery near my heart to go around the area that is blocked." d. "I will need to take an aspirin every day after the surgery to keep the graft open."

ANS: B When the internal mammary artery is used there is no need to have a saphenous vein removed from the leg. The other statements by the patient are accurate and indicate that the teaching has been effective.

15. Following an acute myocardial infarction (AMI), a patient ambulates in the hospital hallway. When the nurse is evaluating the patient's response to the activity, which assessment data would indicate that the exercise level should be decreased? a. Blood pressure (BP) changes from 118/60 to 126/68 mm Hg. b. Oxygen saturation drops from 99% to 95%. c. Heart rate increases from 66 to 92 beats/minute. d. Respiratory rate goes from 14 to 20 breaths/minute.

ANS: C A change in heart rate of more than 20 beats over the resting heart rate indicates that the patient should stop and rest. The increases in BP and respiratory rate, and the slight decrease in oxygen saturation, are normal responses to exercise.

12. Heparin is ordered for a patient with a non-ST-segment-elevation myocardial infarction (NSTEMI). What is the purpose of the heparin? a. Heparin enhances platelet aggregation. b. Heparin decreases coronary artery plaque size. c. Heparin prevents the development of new clots in the coronary arteries. d. Heparin dissolves clots that are blocking blood flow in the coronary arteries.

ANS: C Heparin helps prevent the conversion of fibrinogen to fibrin and decreases coronary artery thrombosis. It does not change coronary artery plaque, dissolve already formed clots, or enhance platelet aggregation.

2. Which nursing intervention will be most effective when assisting the patient with coronary artery disease (CAD) to make appropriate dietary changes? a. Give the patient a list of low-sodium, low-cholesterol foods that should be included in the diet. b. Emphasize the increased risk for heart problems unless the patient makes the dietary changes. c. Help the patient modify favorite high-fat recipes by using monosaturated oils when possible. d. Inform the patient that a diet containing no saturated fat and minimal salt will be necessary.

ANS: C Lifestyle changes are more likely to be successful when consideration is given to the patient's values and preferences. The highest percentage of calories from fat should come from monosaturated fats. Although low-sodium and low-cholesterol foods are appropriate, providing the patient with a list alone is not likely to be successful in making dietary changes. Completely removing saturated fat from the diet is not a realistic expectation. Up to 7% of calories in the therapeutic lifestyle changes (TLC) diet can come from saturated fat. Telling the patient about the increased risk without assisting further with strategies for dietary change is unlikely to be successful.

16. During the administration of the thrombolytic agent to a patient with an acute myocardial infarction (AMI), the nurse should stop the drug infusion if the patient experiences a. bleeding from the gums. b. increase in blood pressure. c. a decrease in level of consciousness. d. a nonsustained episode of ventricular tachycardia.

ANS: C The change in level of consciousness indicates that the patient may be experiencing intracranial bleeding, a possible complication of thrombolytic therapy. Some bleeding of the gums is an expected side effect of the therapy but not an indication to stop infusion of the thrombolytic medication. A decrease in blood pressure could indicate internal bleeding. A nonsustained episode of ventricular tachycardia is a common reperfusion dysrhythmia and may indicate that the therapy is effective.

18. In preparation for discharge, the nurse teaches a patient with chronic stable angina how to use the prescribed short-acting and long-acting nitrates. Which patient statement indicates that the teaching has been effective? a. "I will check my pulse rate before I take any nitroglycerin tablets." b. "I will put the nitroglycerin patch on as soon as I get any chest pain." c. "I will stop what I am doing and sit down before I put the nitroglycerin under my tongue." d. "I will be sure to remove the nitroglycerin patch before taking any sublingual nitroglycerin."

ANS: C The patient should sit down before taking the nitroglycerin to decrease cardiac workload and prevent orthostatic hypotension. Transdermal nitrates are used prophylactically rather than to treat acute pain and can be used concurrently with sublingual nitroglycerin. Although the nurse should check blood pressure before giving nitroglycerin, patients do not need to check the pulse rate before taking nitrates.

17. A patient is recovering from a myocardial infarction (MI) and develops chest pain on day 3 that increases when taking a deep breath and is relieved by leaning forward. Which action should the nurse take next? a. Assess the feet for pedal edema. b. Palpate the radial pulses bilaterally. c. Auscultate for a pericardial friction rub. d. Check the heart monitor for dysrhythmias.

ANS: C The patient's symptoms are consistent with the development of pericarditis, a possible complication of MI. The other assessments listed are not consistent with the description of the patient's symptoms.

4. Which information given by a patient admitted with chronic stable angina will help the nurse confirm this diagnosis? a. The patient states that the pain "wakes me up at night." b. The patient rates the pain at a level 3 to 5 (0 to 10 scale). c. The patient states that the pain has increased in frequency over the last week. d. The patient states that the pain "goes away" with one sublingual nitroglycerin tablet.

ANS: D Chronic stable angina is typically relieved by rest or nitroglycerin administration. The level of pain is not a consistent indicator of the type of angina. Pain occurring at rest or with increased frequency is typical of unstable angina.

8. A patient who has had chest pain for several hours is admitted with a diagnosis of rule out acute myocardial infarction (AMI). Which laboratory test should the nurse monitor to help determine whether the patient has had an AMI? a. Myoglobin b. Homocysteine c. C-reactive protein d. Cardiac-specific troponin

ANS: D Troponin levels increase about 4 to 6 hours after the onset of myocardial infarction (MI) and are highly specific indicators for MI. Myoglobin is released within 2 hours of MI, but it lacks specificity and its use is limited. The other laboratory data are useful in determining the patient's risk for developing coronary artery disease (CAD) but are not helpful in determining whether an acute MI is in progress.

1. The nurse prepares a discharge teaching plan for a 44-yr-old male patient who has recently been diagnosed with coronary artery disease (CAD). Which risk factor should the nurse plan to focus on during the teaching session? A Type A personality B Elevated serum lipids C Family cardiac history D Hyperhomocysteinemia

B Dyslipidemia is one of the four major modifiable risk factors for CAD. The other major modifiable risk factors are hypertension, tobacco use, and physical inactivity. Research findings related to psychologic states (i.e., type A personality) as a risk factor for coronary artery disease have been inconsistent. Family history is a nonmodifiable risk factor. High homocysteine levels have been linked to an increased risk for CAD.

16. A patient returns after cardiac catheterization. Which nursing care would the registered nurse delegate to the licensed practical nurse? A Monitor the electrocardiogram for dysrhythmias B Check for bleeding at the catheter insertion site C Prepare discharge teaching related to complications D Take vital signs and report abnormal values

B The licensed practical nurse can check for bleeding at the puncture sites. If bleeding is identified, it should be reported to the registered nurse. Vital signs should be delegated to the unlicensed assistive personnel. Preparation of discharge teaching and monitoring for dysrhythmias such as S-T elevation would be registered nurse scope of practice.

14. The nurse is examining the electrocardiogram (ECG) of a patient just admitted with a suspected MI. Which ECG change is most indicative of prolonged or complete coronary occlusion? A Sinus tachycardia B Pathologic Q wave C Fibrillatory P waves D Prolonged PR interval

B The presence of a pathologic Q wave, as often accompanies STEMI, is indicative of complete coronary occlusion. Sinus tachycardia, fibrillatory P waves (e.g., atrial fibrillation), or a prolonged PR interval (first-degree heart block) are not direct indicators of extensive occlusion.

8. After teaching a patient with chronic stable angina about nitroglycerin, the nurse recognizes the need for further teaching when the patient makes which statement? A "I will replace my nitroglycerin supply every 6 months." B "I can take up to five tablets every 3 minutes for relief of my chest pain." C "I will take acetaminophen (Tylenol) to treat the headache caused by nitroglycerin." D "I will take the nitroglycerin 10 minutes before planned activity that usually causes chest pain."

B The recommended dose of nitroglycerin is one tablet taken sublingually (SL) or one metered spray for symptoms of angina. If symptoms are unchanged or worse after 5 minutes, the patient should be instructed to activate the emergency medical services (EMS) system. If symptoms are improved, repeat the nitroglycerin every 5 minutes for a maximum of three doses and contact EMS if symptoms have not resolved completely.

4. A 74-yr-old man with a history of prostate cancer and hypertension is admitted to the emergency department with substernal chest pain. Which action will the nurse complete before administering sublingual nitroglycerin? A Administer morphine sulfate IV. B Auscultate heart and lung sounds. C Obtain a 12-lead electrocardiogram (ECG). D Assess for coronary artery disease risk factors.

C If a patient has chest pain, the nurse should institute the following measures: (1) administer supplemental oxygen and position the patient in upright position unless contraindicated, (2) assess vital signs, (3) obtain a 12-lead ECG, (4) provide prompt pain relief first with a nitrate followed by an opioid analgesic if needed, and (5) auscultate heart sounds. Obtaining a 12-lead ECG during chest pain aids in the diagnosis.

12. Postoperative care of a patient undergoing coronary artery bypass graft (CABG) surgery includes monitoring for which common complication? A Dehydration B Paralytic ileus C Atrial dysrhythmias D Acute respiratory distress syndrome

C Postoperative dysrhythmias, specifically atrial dysrhythmias, are common in the first 3 days after CABG surgery. Although the other complications could occur, they are not common complications.

22. The nurse assesses the right femoral artery puncture site as soon as the patient arrives after having a stent inserted into a coronary artery. The insertion site is not bleeding or discolored. What should the nurse do next to ensure the femoral artery is intact? A Palpate the insertion site for induration. B Assess peripheral pulses in the right leg. C Inspect the patient's right side and back. D Compare the color of the left and right legs.

C The best method to determine that the right femoral artery is intact after inspection of the insertion site is to logroll the patient to inspect the right side and back for retroperitoneal bleeding. The artery can be leaking and blood is drawn into the tissues by gravity. The peripheral pulses, color, and sensation of the right leg will be assessed per agency protocol.

17. In caring for the patient with angina, the patient said, "While I was having a bowel movement, I started having the worst chest pain ever, like before I was admitted. I called for a nurse, then the pain went away." What further assessment data should the nurse obtain from the patient? A "What precipitated the pain?" B "Has the pain changed this time?" C "In what areas did you feel this pain?" D "What is your pain level on a 0 to 10 scale?"

C Using PQRST, the assessment data not volunteered by the patient is the radiation of pain, the area the patient felt the pain, and if it radiated. The precipitating event was going to the bathroom and having a bowel movement. The quality of the pain was "like before I was admitted," although a more specific description may be helpful. Severity of the pain was the "worst chest pain ever," although an actual number may be needed. Timing is supplied by the patient describing when the pain occurred and that he had previously had this pain.

24. Which antilipemic medications should the nurse question for a patient with cirrhosis of the liver (select all that apply.)? A Niacin B Cholestyramine C Ezetimibe (Zetia) D Gemfibrozil (Lopid) E Atorvastatin (Lipitor)

C D E Ezetimibe (Zetia) should not be used by patients with liver impairment. Adverse effects of atorvastatin (Lipitor), a statin drug, include liver damage and myopathy. Liver enzymes must be monitored frequently and the medication stopped if these enzymes increase. Niacin's side effects subside with time, although decreased liver function may occur with high doses. Cholestyramine is safe for long-term use.

2. The nurse instructs a 68-yr-old woman with hypercholesterolemia about natural lipid-lowering therapies. The nurse determines further teaching is necessary if the patient makes which statement? A "Omega-3 fatty acids are helpful in reducing triglyceride levels." B "I should check with my physician before I start taking any herbal products." C "Herbal products do not go through as extensive testing as prescription drugs do." D "I will take garlic instead of my prescription medication to reduce my cholesterol."

D Current evidence does not support using garlic in the treatment of elevated cholesterol. Strong evidence supports the use of omega-3 fatty acids for reduction of triglyceride levels. Many herbal products are not standardized and effects are not predictable. Patients should consult with their health care provider before starting herbal or natural therapies.

13. A patient admitted to the emergency department 24 hours ago with complaints of chest pain was diagnosed with a ST-segment-elevation myocardial infarction (STEMI). What complication of myocardial infarction should the nurse anticipate? A Unstable angina B Cardiac tamponade C Sudden cardiac death D Cardiac dysrhythmias

D Dysrhythmias are present in 80% to 90% of patients after myocardial infarction (MI). Unstable angina is considered a precursor to MI rather than a complication. Cardiac tamponade is a rare event, and sudden cardiac death is defined as an unexpected death from cardiac causes. Cardiac dysfunction in the period following an MI would not be characterized as sudden cardiac death.

20. A female patient who has type 1 diabetes mellitus has chronic stable angina that is controlled with rest. She states that over the past few months, she has required increasing amounts of insulin. What goal should the nurse use to plan care that should help prevent cardiovascular disease progression? A Exercise almost every day. B Avoid saturated fat intake. C Limit calories to daily limit. D Keep Hgb A1C (A1C) less than 7%.

D If the Hgb A1C (A1C) is kept below 7%, this means that the patient has had good control of her blood glucose over the past 3 months. The patient indicates that increasing amounts of insulin are being required to control her blood glucose. This patient may not be adhering to the dietary guidelines or therapeutic regimen, so teaching about how to maintain diet, exercise, and medications to maintain stable blood glucose levels will be needed to achieve this goal.

18. The patient is being dismissed from the hospital after acute coronary syndrome and will be attending rehabilitation. What information would be taught in the early recovery phase of rehabilitation? A Therapeutic lifestyle changes should become lifelong habits. B Physical activity is always started in the hospital and continued at home. C Attention will focus on management of chest pain, anxiety, dysrhythmias, and other complications. D Activity level is gradually increased under cardiac rehabilitation team supervision and with electrocardiographic (ECG) monitoring.

D In the early recovery phase after the patient is dismissed from the hospital, the activity level is gradually increased under supervision and with ECG monitoring. The late recovery phase includes therapeutic lifestyle changes that become lifelong habits. In the first phase of recovery, activity is dependent on the severity of the angina or myocardial infarction, and attention is focused on the management of chest pain, anxiety, dysrhythmias, and other complications. With early recovery phase, the cardiac rehabilitation team may suggest that physical activity be initiated at home, but this is not always done.

15. For which problem is percutaneous coronary intervention (PCI) most clearly indicated? A Chronic stable angina B Left-sided heart failure C Coronary artery disease D Acute myocardial infarction

D PCI is indicated to restore coronary perfusion in cases of myocardial infarction. Chronic stable angina and coronary artery disease are normally treated with more conservative measures initially. PCI is not relevant to the pathophysiology of heart failure.


Related study sets

Biology IGCSE - Coordination and Response

View Set

Psychology 375 Cognition - Final

View Set

networking threats, assessments, and defenses

View Set

GOVT2305 - U.S. Government - Chapter 4

View Set